Đến nội dung

Vitamin Love

Vitamin Love

Đăng ký: 22-07-2019
Offline Đăng nhập: 30-07-2019 - 19:09
-----

Trong chủ đề: $ \boxed{TOPIC} $ Các bài toán hình học hướng đế...

29-07-2019 - 21:52

Bài toán 2 còn có 1 cách giải sử dụng đường trung bình. 

Kẻ đường kính $ AK $ của $ (O) $. Gọi $ H $ là trực tâm $ \Delta AMN $, $ T $ là trung điểm $ HK $.

Dễ thấy $  NH // BK $ ( cùng $ \bot AB $ ) mà $ P, T $ là trung điểm $ BN, HK $ nên $ NH // PT // BK $ 

Mặt khác $ IP // AB $ suy ra $ PT \bot IP $. 

Chứng minh tương tự ta có $  OT \bot MN $ , $ IQ \bot QT $ 

Vậy 5 điểm $ I, O , Q , T, P $ đồng viên. 

Suy ra điều phải chứng minh. 

Mở rộng : $ \Delta ABC $, $ d $ bất kì đi qua $ ( O ) $ cắt   $ ( O ) , AB, AC $ lần lượt tại $ X, Y, M, N $. Gọi $ J, I , P , Q $ là trung điểm $ XY, MN, BN, CM $. Khi đó $ JINM $ nội tiếp. 

 

 

( P/S mình bổ sung thêm qui định đó là sau 3 ngày bài toán đề xuất không có lời giải thì người up có thể up lời giải cho mn cùng tham khảo, tránh tình trạng bí cục bộ  :lol:, người up vẫn sẽ đề xuất bài tiếp theo. )

Trung điểm XY ?? PQ có bị thừa ??


Trong chủ đề: TOPIC Tổ hợp-Xác suất

25-07-2019 - 21:30

Cho e xin đề xuất bài sau: Cho cái đồng hồ ở mỗi số đặt một bóng đèn. Ban đầu bóng 1 sáng còn lại tắt. Mỗi lần chọn 3 bóng đèn liên tiếp và thay đổi trạng thái của nó. Hỏi có thể thay đổi sao cho bóng số 2 sáng còn lại tắt được hay không?


Trong chủ đề: $ \boxed{TOPIC} $ Các bài toán hình học hướng đế...

23-07-2019 - 16:32

AP cắt $(I)$ tại K. Q đối xứng với K qua AM. FP cắt $(I)$ tại $R$

Do $AM.AI=AK.AP=AD^2$ nên KMIP nội tiếp. Do đó $\widehat{KMP}=\widehat{KIP}$ suy ra $\widehat{KMP}+\widehat{KMA}+\widehat{AMQ}=\widehat{KIP}+2\widehat{KPI}=180^0$ nên P, M, Q thẳng hàng.

Do vậy nên ta có: $\frac{DM}{MP}=\frac{ME}{MP}=\frac{QE}{DP}=\frac{DK}{DP}=\frac{DA}{PA}=\frac{DA}{FP}$ (1)

Mặt khác,do F đối xứng với A qua DP nên DP là phân giác $\widehat{RPK}$ suy ra $DR=DK=QE$ và $DE=RQ$

Do đó: $\widehat{RPM}=\widehat{ADM}$ (2)

Từ (1) và (2) suy ra tam giác FPM đồng dạng với tam giác ADM nên $\widehat{FMP}=\widehat{AMD}=90^0$.

Bài 2: Cho tam giác $ABC$ và $(O)$ là đường tròn ngoại tiếp. Một đường thẳng d bất kỳ đi qua $O$ cắt $AB,AC$ lần lượt tại $M$ và $N$. Gọi $I,P,Q$ lần lượt là trung điểm của $MN,BN,CM$. Chứng minh 4 điểm $O,I,P,Q$ cùng nằm trên một đường tròn.

P/s: Ai biết xuất xứ bài toán này không cho mình biết với....

 

 

AP cắt $(I)$ tại K. Q đối xứng với K qua AM. FP cắt $(I)$ tại $R$

Do $AM.AI=AK.AP=AD^2$ nên KMIP nội tiếp. Do đó $\widehat{KMP}=\widehat{KIP}$ suy ra $\widehat{KMP}+\widehat{KMA}+\widehat{AMQ}=\widehat{KIP}+2\widehat{KPI}=180^0$ nên P, M, Q thẳng hàng.

Do vậy nên ta có: $\frac{DM}{MP}=\frac{ME}{MP}=\frac{QE}{DP}=\frac{DK}{DP}=\frac{DA}{PA}=\frac{DA}{FP}$ (1)

Mặt khác,do F đối xứng với A qua DP nên DP là phân giác $\widehat{RPK}$ suy ra $DR=DK=QE$ và $DE=RQ$

Do đó: $\widehat{RPM}=\widehat{ADM}$ (2)

Từ (1) và (2) suy ra tam giác FPM đồng dạng với tam giác ADM nên $\widehat{FMP}=\widehat{AMD}=90^0$.

Bài 2: Cho tam giác $ABC$ và $(O)$ là đường tròn ngoại tiếp. Một đường thẳng d bất kỳ đi qua $O$ cắt $AB,AC$ lần lượt tại $M$ và $N$. Gọi $I,P,Q$ lần lượt là trung điểm của $MN,BN,CM$. Chứng minh 4 điểm $O,I,P,Q$ cùng nằm trên một đường tròn.

P/s: Ai biết xuất xứ bài toán này không cho mình biết với....

 

Kẻ đường kính BE CF.
Ta có NE // OP  MF // OQ
=> ∠EKF = ∠POQ (1)
Lại có K thuộc (O) theo định lí pascal ( K là giao của EN và FM )
=> ∠EKF = ∠BAC = ∠PIQ (2)
Từ (1) và (2) => ∠PIQ = ∠EKF => ĐPMC
( bài này xuất phát từ IMO 2009 )
Bài toán 3: Cho tam giác ABC nhọn nt (O) đường tròn (K) đi qua BC cắt AC AB tại E F, BE CF cắt tại H. P là điểm tùy ý trên AH. (APB) (APC) cắt BE CF tại M, N. X, Y, Z theo thứ tự là tâm đường tròn (APB) (APC) (BMC). Q, R là tâm (XYZ) (MNP). Chứng minh: QR // AP, (XYZ) (MNP) đồng tâm (khi K thuộc BC)